PT3.S2.Q4 - A work of architecture, if it is to be both

JaytrongJaytrong Alum Member
edited May 2019 in Logical Reasoning 54 karma

Hi, I am having trouble understanding why (B) MBT. I understand why the other answer choices are bad. But I have no idea why (B) is the correct answer. It seems to me that (B) CBT -- not MBT.

My diagram is as follows:
Old Precept: Inviting & Functional --> unobtrusive
New Precept: Inviting & Functional --> /unobtrusive
Modern Architects --> Strong Personality --> /Functional

So going by the new precept, I do not see how it logically follows that Modern Architects --> Strong Personality --> /unobtrusive.

Admin note: edited title

Comments

  • BlindReviewerBlindReviewer Alum Member
    855 karma

    I was also confused -- I think this ultimately comes down to negating a conditional statement. When they say the precept was violated, we can take that to mean the conditional statement is negated: if the original statement is A -> B, then A -> /B would be the negation.

    So in this case, Inviting & Functional -> unobtrusive must be Inviting & Functional -> /unobtrusive.

    The weird part is just that the stimulus says the building is actually NOT Inviting & Functional. But I think regardless of whether or not the sufficient condition is satisfied, the necessary cannot be satisfied for the precept to be violated? If /Inviting & Functional -> unobtrusive, that would not be violating the precept at all, because if the sufficient is failed then the rule falls away. So in either case, whether the sufficient is met or failed, we need to negate the necessary as /unobtrusive.

    I think that makes sense (kind of just worked through it while typing this up) but definitely convoluted and I haven't seen anything like this on recent LSATs so it might not be worth dwelling on!

  • LSATcantwinLSATcantwin Alum Member Sage
    edited May 2019 13286 karma

    *edited out confusing explanation

    Clarified below

  • JaytrongJaytrong Alum Member
    edited May 2019 54 karma

    @LSATcantwin, No offense, but your explanation is inconsistent... You first claim " IF inviting AND functional THEN unobtrusive"Then you follow that up with the statement "You need both I and f in order to get to unobtrusive."This is diagrammed as
    unobtrusive ---> inviting and functional
    Clearly, you're confusing necessity and sufficiency since you end your explanation with "not unobtrusive --> not inviting or not functional." Provided as such, I have no idea how you derived your conclusion.

    In regards to violating the precept, i don't really understand what you mean. Going against a conditional, is negating a conditional... If this is the case, we get either

    (a) Inviting and Functional --> not unobtrusive; or
    (b) unobtrustive --> ~inviting or ~functional.

    We have ~functional. With the sufficient condition failed, we cannot properly infer the necessary. If this is the case, why is answer choice (A) a MUST BE TRUE answer; when it seems to be the case that it is COULD BE TRUE.

  • JaytrongJaytrong Alum Member
    edited May 2019 54 karma

    @BlindReviewer I think you might be on to something; but so i understand it clearer, do you mind explaining it me with the following example?

    Previously, in order to get a high LSAT score and get into Yale, students must study hard.
    This rule is no longer true. To get a high LSAT score and get into Yale, students do not need to study hard. Current students let their strong extracurricular activities take over their application, submitting applications that not have a high LSAT score.
    If it is agreeable that the above is like the stimulus, then what MBT should, likewise be the following:

    Current students who let their strong extra curricular activities take over their application do not need study hard.

    The reason, you claim, is because whether the sufficient condition is failed, since the rule is no longer true (If students get a high LSAT score and get into Yale, students must study hard), it must follow that current students who let their strong extra curricular activities take over their application do not need study hard -- otherwise, the rule is still true????

  • LSATcantwinLSATcantwin Alum Member Sage
    edited May 2019 13286 karma

    I’m not confusing sufficient and necessary, they are. That’s why they are violating the precept!

    Maybe the clearer way to say it would just be to say that the original statement;

    •IF inviting and functional THEN unobtrusive.

    No longer exists because it has now been violated.

    To violate this means to have the necessary condition no longer be the case. Which means their work IS not unobtrusive.

  • JaytrongJaytrong Alum Member
    edited May 2019 54 karma

    @LSATcantwin
    Wait im super confused now... I agree with what you said in the latter half. That's similar to what @BlindReviewer explained to me, I think. But im confused on your the definition of violating the precept. You said violating the precept is confusing sufficient and necessary in the former half; but in the latter half you said violating the precept is failing the necessary. Can you expand on what you mean by they're confusing sufficiency and necessity?
    Since its a MBT question, if they confuse sufficiency and necessity, we wouldn't have been able to derive an answer choice.

    In regards to the latter half, I'm still slightly confused because from what you said I can only derive (i) A work of architecture of modern architects, who build inviting and functional buildings, are not unobtrusive. But I can't seem to understand how this is derived: (ii) A work of architecture of modern architects, who build non-functional buildings, are not unobtrusive.

  • drbrown2drbrown2 Alum Member
    2227 karma

    Violating precept means violating the rule that it must be unobtrusive
    Argument goes like this:

    If A is both I and FPU, then it must be unobtrusive (2nd to environment).
    Modern architects (who are all plagued by egoism) have violated rule that it must be unobtrusive.
    They let strong personalities take over work, resulting in A that is not FPU.

    So they negated the necessary condition, which means that either I or FPU would be negated. They chose FPU. What else is provable? Either a simple restatement of the conditional premises, or the correct answer which basically says all modern architects who let personalities take over work violate the rule that it must be unobtrusive. Modern architects that let their strong personalities take over their work producing buildings that are not FPU definitely fall into the category of modern architects who violated the rule.

  • JaytrongJaytrong Alum Member
    54 karma

    @drbrown2 Yeah... I honestly have no clue what you're saying with this: "So they negated the necessary condition, which means that either I or FPU would be negated. They chose FPU." Lets start at the top.

    (i) Unviolated Precept: Inviting and Functional --> Unobtrusive
    (ii) Unviolated Precept Contrapositive: Not unobtrusive --> NOT Inviting OR NOT Functional

    (iii) Violated Precept: Inviting and Functional --> NOT Unobtrusive
    (iv) Violated Precept Contrapositive: Unobtrusive --> NOT Inviting OR NOT Functional

    We are given NOT Functional.

    Using what rule, can we infer that the AC -- Modern Architects with strong personalities produce buildings that are Unobtrusive -- is MBT?

  • JaytrongJaytrong Alum Member
    edited May 2019 54 karma

    @BlindReviewer @LSATcantwin @drbrown2

    It took me a while, but tell me what you think of the following explanation. It seems to make waaay more sense to me. In essence, the answer to is irrelevant to whether the precept is violated. That was just a side point, made to distract us.

    Explanation:

    The definition of unobtrusive is work of architect takes 2nd place to total environment.

    We know that Modern architects with strong personalities let their personalities take over their work. If this is the case, then the hierarchy, now, can be something like the following:
    1. Total Environment
    2. Strong personality
    3. Work of architect

    If this is the new hierarchy, then Modern Architects with strong personalities produce work that are NOT Unobtrusive.

    So, rather than using any formal logic to derive the answer, the answer was explicitly stated. We just had to equate and understand the terms. In other words, AC (B) is a restatement of "They have let their strong personalities take over their work."

    help

  • drbrown2drbrown2 Alum Member
    2227 karma

    @Jaytrong So what I meant by the shorthand and brief description of what the argument did was this...

    A precept is basically a rule or a mandate. The second sentence from the premise states that modern architects have "violated this precept" which is referential phrasing for "must be unobtrusive." This basically means they negated the necessary condition stated in the first sentence.
    They run out the entire contrapositive chain and state that their personalities take over and they produce buildings that are not functional for public use.

    Your most recent post about the formal logic being irrelevant is correct because you don't need to do any contrapositives to get to the correct answer. You just need to think of all modern architects as a set of people who all violate the rule/precept that buildings must be unobtrusive. In the last sentence, it says that "they (referential for modern architects) let their strong personalities take over." So you are absolutely correct that you just need to equate terms, and parse out the referential phrasing to prove out AC B.

  • zheng18552zheng18552 Free Trial Member
    24 karma

    This questions is really odd. I solved this by POE and subjective sense of what is more likely to be wrong. I hazard formal logic cannot solve this issue without making too many assumptions

  • zheng18552zheng18552 Free Trial Member
    edited November 2019 24 karma

    1: I(inviting) and F( fuctionional) ----> U(unobtrusive)

    2: U=taking 2nd place to the environment

    3: SPTOW(strong personality taking over the work) -> not F

    B ) is saying SPTOW-> U

    1 can be negated as not U -> not I or not F

    well, not F does not necessarily mean not U. that is an illegal negation.

    Unless one makes certain assumptions, overstretching in my view, one cannot solve this questions by formal logic. Hence, so much heated discussion if not quarrels. For instance, an violation of a percept does not necessarily mean the percept is violated in the exact way of I and F --> not Unobtrusive.

    I'd say, just go with POE and use your gut feelings.

Sign In or Register to comment.